Difference between revisions of "2024 AMC 12B Problems/Problem 8"
m (→Solution 3 (Using Variables)) |
|||
Line 42: | Line 42: | ||
Then, from our definitions of <math>a</math> and <math>b</math>, <math>2^a=x</math> and <math>3^b=x</math>, so <math>2^a=3^b.</math> Taking the logarithm base <math>3</math> of both sides of this equation gives us <math>\log_3 2^a=b</math>, hence <math>a \log_3 2=b.</math> | Then, from our definitions of <math>a</math> and <math>b</math>, <math>2^a=x</math> and <math>3^b=x</math>, so <math>2^a=3^b.</math> Taking the logarithm base <math>3</math> of both sides of this equation gives us <math>\log_3 2^a=b</math>, hence <math>a \log_3 2=b.</math> | ||
Now, we substitute <math>a \log_3 2</math> for <math>b</math> in the equation, which gives <cmath>\frac{a \cdot a \log_3 2}{a+a \log_3 2}=2.</cmath>Notice that we can factor out an <math>a</math> in the numerator and denominator, if <math>a \neq 0,</math> and doing so yields <cmath>\frac{a \log_3 2}{1+\log_3 2}=2.</cmath> We know that <math>1= \log_3 3,</math> so putting that in gives us <cmath>\frac{a \log_3 2}{\log_3 3+\log_3 2}=2 \implies \frac{a \log_3 2}{\log_3 6}=2.</cmath>So, <math>a=2 \cdot \frac{\log_3 6}{\log_3 2}</math>, which, using the change of base formula in reverse, is equivalent to <math>2 \cdot \log_2 6,</math> thus, <cmath>a= \log _2 6^2= \log _2 36.</cmath> Finally, using our original definition of <math>a,</math> we have <cmath>a = \log_2 x=\log_2 36,</cmath> so <math>x=\boxed{\textbf{(C) }36}.</math> | Now, we substitute <math>a \log_3 2</math> for <math>b</math> in the equation, which gives <cmath>\frac{a \cdot a \log_3 2}{a+a \log_3 2}=2.</cmath>Notice that we can factor out an <math>a</math> in the numerator and denominator, if <math>a \neq 0,</math> and doing so yields <cmath>\frac{a \log_3 2}{1+\log_3 2}=2.</cmath> We know that <math>1= \log_3 3,</math> so putting that in gives us <cmath>\frac{a \log_3 2}{\log_3 3+\log_3 2}=2 \implies \frac{a \log_3 2}{\log_3 6}=2.</cmath>So, <math>a=2 \cdot \frac{\log_3 6}{\log_3 2}</math>, which, using the change of base formula in reverse, is equivalent to <math>2 \cdot \log_2 6,</math> thus, <cmath>a= \log _2 6^2= \log _2 36.</cmath> Finally, using our original definition of <math>a,</math> we have <cmath>a = \log_2 x=\log_2 36,</cmath> so <math>x=\boxed{\textbf{(C) }36}.</math> | ||
+ | |||
+ | ~hdanger | ||
==See also== | ==See also== | ||
{{AMC12 box|year=2024|ab=B|num-b=7|num-a=9}} | {{AMC12 box|year=2024|ab=B|num-b=7|num-a=9}} | ||
{{MAA Notice}} | {{MAA Notice}} |
Revision as of 12:42, 18 November 2024
Contents
Problem
What value of satisfies
Solution 1
We have \begin{align*} \log_2x\cdot\log_3x&=2(\log_2x+\log_3x) \\ 1&=\frac{2(\log_2x+\log_3x)}{\log_2x\cdot\log_3x} \\ 1&=2(\frac{1}{\log_3x}+\frac{1}{\log_2x}) \\ 1&=2(\log_x3+\log_x2) \\ \log_x6&=\frac{1}{2} \\ x^{\frac{1}{2}}&=6 \\ x&=36 \end{align*} so
~kafuu_chino
Solution 2 (Change of Base)
\begin{align*} \frac{\log_2x \cdot \log_3x}{\log_2x+\log_3x} &= 2 \\[6pt] \log_2x \cdot \log_3x &= 2(\log_2x+\log_3x) \\[6pt] \log_2x \cdot \log_3x &= 2\log_2x + 2\log_3x \\[6pt] \frac{\log x}{\log 2} \cdot \frac{\log x}{\log 3} &= 2\frac{\log x}{\log 2} + 2\frac{\log x}{\log 3} \\[6pt] \frac{(\log x)^2}{\log 2 \cdot \log 3} &= \frac{2\log x \cdot \log 3 + 2\log x \cdot \log 2}{\log 2 \cdot \log 3} \\[6pt] (\log x)^2 &= 2\log x \cdot \log 3 + 2\log x \cdot \log 2 \\[6pt] (\log x)^2 &= 2\log x(\log 2 + \log 3) \\[6pt] \log x &= 2(\log 2 + \log 3) \\[6pt] x &= 10^{2(\log 2 + \log 3)} \\[6pt] x &= (10^{\log 2} \cdot 10^{\log 3})^2 \\[6pt] x &= (2 \cdot 3)^2 = 6^2 = \boxed{\textbf{(C) }36} \end{align*}
~sourodeepdeb
Solution 3 (Using Variables)
Let and . This gives us the equation
Then, from our definitions of and , and , so Taking the logarithm base of both sides of this equation gives us , hence Now, we substitute for in the equation, which gives Notice that we can factor out an in the numerator and denominator, if and doing so yields We know that so putting that in gives us So, , which, using the change of base formula in reverse, is equivalent to thus, Finally, using our original definition of we have so
~hdanger
See also
2024 AMC 12B (Problems • Answer Key • Resources) | |
Preceded by Problem 7 |
Followed by Problem 9 |
1 • 2 • 3 • 4 • 5 • 6 • 7 • 8 • 9 • 10 • 11 • 12 • 13 • 14 • 15 • 16 • 17 • 18 • 19 • 20 • 21 • 22 • 23 • 24 • 25 | |
All AMC 12 Problems and Solutions |
The problems on this page are copyrighted by the Mathematical Association of America's American Mathematics Competitions.